Solving For Sides II

Algebra Level 4

The equation

( x 5 ) ( x 12 ) ( x 13 ) = 20 (x-5)(x-12)(x-13)=20

has three real solutions, r 1 , r_1, r 2 , r_2, and r 3 . r_3. If a triangle with sides of r 1 , r_1, r 2 , r_2, and r 3 r_3 has area A , A, give A 2 . A^2.


The answer is 600.

This section requires Javascript.
You are seeing this because something didn't load right. We suggest you, (a) try refreshing the page, (b) enabling javascript if it is disabled on your browser and, finally, (c) loading the non-javascript version of this page . We're sorry about the hassle.

4 solutions

Jon Haussmann
May 9, 2014

More generally, let a a , b b , and c c be the roots of f ( x ) = x 3 + p x 2 + q x + r , f(x) = x^3 + px^2 + qx + r, so f ( x ) = ( x a ) ( x b ) ( x c ) f(x) = (x - a)(x - b)(x - c) .

Assuming that a a , b b , and c c form the sides of a triangle, by Heron's formula, K 2 = s ( s a ) ( s b ) ( s c ) , K^2 = s(s - a)(s - b)(s - c), where K K and s s are the area and semi-perimeter of the triangle, respectively.

By Vieta's formulas, s = ( a + b + c ) / 2 = p / 2 s = (a + b + c)/2 = -p/2 , and f ( p 2 ) = f ( s ) = ( s a ) ( s b ) ( s c ) . f \left( -\frac{p}{2} \right) = f(s) = (s - a)(s - b)(s - c). Hence, K 2 = s ( s a ) ( s b ) ( s c ) = p 2 f ( p 2 ) . K^2 = s(s - a)(s - b)(s - c) = -\frac{p}{2} f \left( -\frac{p}{2} \right).

Here, f ( x ) = ( x 5 ) ( x 12 ) ( x 13 ) 20 f(x) = (x - 5)(x - 12)(x - 13) - 20 , and p = ( 5 + 12 + 13 ) = 30 p = -(5 + 12 + 13) = -30 , so K 2 = 15 [ ( 15 5 ) ( 15 12 ) ( 15 13 ) 20 ] = 600. K^2 = 15 [(15 - 5)(15 - 12)(15 - 13) - 20] = 600.

Follow-up Question: Is there an easy way to tell that the roots of f ( x ) = ( x 5 ) ( x 12 ) ( x 13 ) 20 f(x) = (x - 5)(x - 12)(x - 13) - 20 do form the sides of a triangle?

Good follow-up question. For an observation, ( x 5 ) ( x 12 ) ( x 13 ) 60 = ( x 15 ) ( x 8 ) ( x 7 ) (x-5)(x-12)(x-13) - 60 = (x-15)(x-8)(x-7) , which seems to be the limit when the roots no longer form a triangle.

Calvin Lin Staff - 7 years, 1 month ago

To answer your follow-up question,that is, for what positive values n n does the roots of f ( x ) = ( x 5 ) ( x 12 ) ( x 13 ) n f(x)=(x-5)(x-12)(x-13)-n form a triangle, we first assume WLOG that the roots are r 1 < r 2 < r 3 r_1<r_2<r_3 .

Since r 1 + r 2 + r 3 = 30 r_1+r_2+r_3=30 , hence the roots are sides of a triangle r 1 + r 2 > r 3 30 r 3 > r 3 r 3 < 15 \iff r_1+r_2>r_3\iff 30-r_3>r_3\iff r_3<15 , in other words, when the largest root of f f is less than 15 15 .

Visually, we know that as n n increases, the graph of f f moves down wrt the graph of ( x 5 ) ( x 12 ) ( x 13 ) (x-5)(x-12)(x-13) , thus increasing the largest root, this tells us that n n has an upper bound, which occurs when the largest root takes on 15 ( 15 5 ) ( 15 12 ) ( 15 13 ) n = 0 n = 60 15 \iff (15-5)(15-12)(15-13)-n=0\Rightarrow n=60 . Hence n < 60 n<60 is our answer, as stated by Calvin..

Xuming Liang - 7 years, 1 month ago

Assuming that the sides of the triangle are a , b , c a, b, c with a b c > 0 a \ge b \ge c > 0 , a , b , c a, b, c form a triangle if and only if b + c > a b + c > a . That means a + b + c > 0 -a + b + c > 0 and 2 s 2 a > 0 2s - 2a > 0 and s a > 0 s - a > 0 where s = a + b + c 2 s = \frac{a + b + c}{2} . We know that since a c a \ge c and since b > 0 b > 0 , we have a + b > c a + b >c and thus s c > 0 s - c > 0 even when a , b , c a, b, c do not form a triangle. Similarly, s b > 0 s - b > 0 even when a , b , c a, b, c do not form a triangle. We know that since a , b , c > 0 a, b, c > 0 , we have a + b + c > 0 a + b + c > 0 and thus a + b + c 2 > 0 \frac{a + b + c}{2} > 0 and thus s > 0 s > 0 even when a , b , c a, b, c do not form a triangle. Multiplying our inequalities together, we have ( s ) ( s a ) ( s b ) ( s c ) > 0 (s)(s - a)(s - b)(s - c) > 0 if and only if a , b , c a, b, c form a triangle. But the expression ( s ) ( s a ) ( s b ) ( s c ) (s)(s - a)(s - b)(s - c) is under a square root in Heron's Formula for the area of a triangle, so the only way Heron's formula will give a positive area is if a , b , c a, b, c form a triangle. If they do not, you will get either 0 or a non-real area.

Colin Tang - 7 years, 1 month ago
Pi Han Goh
May 9, 2014

Expanding and simplying gives x 3 30 x 2 + 281 x 800 = 0 x^3 - 30x^2 + 281x - 800 = 0 . Let f ( x ) = x 3 30 x 2 + 281 x 800 f(x) = x^3 - 30x^2 + 281x - 800 , then its zeros are r 1 , r 2 , r 3 r_1, r_2, r_3 .

Similarly, f ( x ) = 0 f(\sqrt x) = 0 has zeros r 1 2 , r 2 2 , r 3 2 r_1^2, r_2^2, r_3^2

x x 30 x + 281 x 800 = 0 x ( x + 281 ) 2 = ( 30 x + 800 ) 2 x \sqrt{x} - 30x + 281 \sqrt{x} - 800 = 0 \Rightarrow x(x+281)^2 = (30x+800)^2

x 3 338 x 2 + 30961 x 640000 = 0 \Rightarrow x^3 - 338x^2 + 30961x - 640000 = 0

r 1 2 + r 2 2 + r 3 2 = 338 , r 1 2 r 2 2 + r 1 2 r 3 2 + r 2 2 r 3 2 = 30961 \Rightarrow r_1^2 + r_2^2+ r_3^2 = 338, r_1^2 r_2^2 + r_1^2 r_3^2 + r_2^2 r_3^2 = 30961

r 1 4 + r 2 4 + r 3 4 = ( r 1 2 + r 2 2 + r 3 2 ) 2 2 ( r 1 2 r 2 2 + r 1 2 r 3 2 + r 2 2 r 3 2 ) = 33 8 2 2 30961 \Rightarrow r_1^4 + r_2^4 + r_3^4 = (r_1^2 + r_2^2+ r_3^2)^2 - 2 ( r_1^2 r_2^2 + r_1^2 r_3^2 + r_2^2 r_3^2) = 338^2 - 2 \cdot 30961

One of the variations of Heron's formula is A = 1 4 ( a 2 + b 2 + c 2 ) 2 2 ( a 4 + b 4 + c 4 ) A = \frac {1}{4} \sqrt{ (a^2 + b^2+c^2)^2 - 2(a^4 + b^4 + c^4)}

A 2 = 1 16 ( 33 8 2 2 ( 33 8 2 2 30961 ) ) = 600 \Rightarrow A^2 = \frac {1}{16} \cdot \left ( 338^2 - 2 (338^2 - 2 \cdot 30961) \right ) = \boxed{600}

Great, but too big calculations....

Vighnesh Raut - 7 years, 1 month ago
Patrick Corn
May 9, 2014

Let f ( x ) = ( x 5 ) ( x 12 ) ( x 13 ) 20 f(x) = (x-5)(x-12)(x-13)-20 . Then Heron's formula says that A 2 = s ( s r 1 ) ( s r 2 ) ( s r 3 ) A^2 = s(s-r_1)(s-r_2)(s-r_3) where s = 1 2 ( r 1 + r 2 + r 3 ) . s = \frac12(r_1+r_2+r_3). . By Vieta, s = 15 s = 15 , and ( s r 1 ) ( s r 2 ) ( s r 3 ) = f ( s ) = f ( 15 ) (s-r_1)(s-r_2)(s-r_3) = f(s) = f(15) . So in both problems, the answer is 15 f ( 15 ) 15 f(15) . In this problem it equals 600 \fbox{600} .

Outstanding! Your solution explains why one number makes a large difference.

Colin Tang - 7 years, 1 month ago

Hi, I solved this question and got the same answer using vieta's formulas, but I made use of all the three...How did you guys says that f(s) = (s-a)(s-b)(s-c)

gaurav pathak - 7 years, 1 month ago

Log in to reply

Well, that is just a variation

King Zhang Zizhong - 7 years, 1 month ago

lol I used the exact same method

King Zhang Zizhong - 7 years, 1 month ago

Can use graphing calculator-( I have TI-83 +)- and find all three roots. Heron's formula gives us the result. We draw the given curve and y=0 to find intersection through calc function.

0 pending reports

×

Problem Loading...

Note Loading...

Set Loading...